LSAT and Law School Admissions Forum

Get expert LSAT preparation and law school admissions advice from PowerScore Test Preparation.

 Administrator
PowerScore Staff
  • PowerScore Staff
  • Posts: 8916
  • Joined: Feb 02, 2011
|
#82052
Complete Question Explanation

The correct answer choice is (A).

Answer choice (A): This is the correct answer choice.

Answer choice (B):

Answer choice (C):

Answer choice (D):

Answer choice (E):


This explanation is still in progress. Please post any questions below!
 cindymoon14
  • Posts: 4
  • Joined: Mar 06, 2018
|
#45806
A was a contender, but I chose E because I thought it accurately describes what is happening at the end of the passage around 58, where author mentions stealing thunder's potential negative effect, that it "forms a cognitive framework for jurors, who then filter subsequent information through this schema." Doesn't this suggest that the trying to positively frame very harmful information can sometimes be damaging and less effective than, say, "creating credibility in some cases" as stated in answer choice E?

Could you please clarify why E is incorrect? Thanks!
 Daniel Stern
PowerScore Staff
  • PowerScore Staff
  • Posts: 81
  • Joined: Feb 07, 2018
|
#45828
Hi Cindy:

E goes way too far afield of what is supported by the passage, whereas A stops right where it should.

You correctly assessed that the line 58 reference to "cognitive framework" means that information jurors get early in the case will influence how they think about information they receive later. And answer choice A says just that.

But E goes further, and thus cannot be our answer, by comparing the "stealing thunder" technique to some other technique, "creating credibility," which is not discussed in the passage. We only know that stealing thunder sometimes fails as a technique; we can't say that some other technique might be better.

I hope that helps!
Dan

Get the most out of your LSAT Prep Plus subscription.

Analyze and track your performance with our Testing and Analytics Package.